(03.02 MC) The figure shows the location of three points around a lake. The length of the lake, BC, is also shown. (The figure is not drawn to scale.) A 6 mi 2 mi 00 B С Which of the following choices is closest to the distance (in miles) between points A and B?​

(03.02 MC) The Figure Shows The Location Of Three Points Around A Lake. The Length Of The Lake, BC, Is

Answers

Answer 1

9514 1404 393

Answer:

  (b)  5.66 mi

Step-by-step explanation:

You only need to eliminate the unreasonable answers to make the correct choice.

The hypotenuse of the right triangle is its longest side, so the unknown length must be less than 6. That eliminates the last two choices.

The unknown length cannot be less than the difference of the sides shown, so the unknown length must be more than 4. That eliminates the first choice.

The only reasonable choice is ...

  B.  5.66 miles

_____

If you like, you can calculate the distance using the Pythagorean theorem.

  AB² + 2² = 6²

  AB = √(36 -4) = √32 ≈ 5.66 . . . miles


Related Questions

PLEASE HELP! I"LL MARK BRAINLIEST!!!!


△ABC is translated left 3 units to form the image ∆A′B′C′.

What are the coordinates of the vertices of ∆A′B′C′ ?

Enter your answer by filling in the boxes.

A′ =

B′ =

C′ =

Answers

Answer:

a is -1, -2

b is -1, 2

c is 2, 2

Step-by-step explanation:

i hope i helped

Find the volume of a square based pyramid with a base length of 8 m and a height of 14 m.

Answers

[tex]V = \frac{1}{3} \times base \: area \: \times height \\ [/tex]

[tex]V = \frac{1}{3} \times {8}^{2} \times 14 \\ [/tex]

[tex]V = \frac{1}{3} \times 64 \times 14 \\ [/tex]

[tex]V = \frac{896}{3} \: \: \: {m}^{3} \\ [/tex]

3^4x-1=15
Need help with this log question

Answers

Answer:

x=16/81

Step-by-step explanation:

3^4x-1=15

81x-1=15

81x=15+1

81x+16

divide both sides by 81

x=16/81

What is the equation of a line that is parallel to the line 2x + 5y = 10 and passes through the point (–5, 1)? Check all that apply. In addition to the Privileges and Immunities Clause, Article IV, Section 2 includes the

Answers

Hello!

-----------------------------------------------------------------------------------------------------------------

2x+5y=10

We want to turn this equation into y=mx+b form:

5y=-2x+10

Divide both sides by 5:

y=-2/5x+2

Now, the equation of the line parallel to it:

Remember, parallel lines have the same slope.

So these two lines have the same slope (-2/5)

Now, use Point-Slope form:

y-y1=m(x-x1)

y=1=-2/5(x-(-5)

y-1=-2/5(x+5)

y-1=-2/5x-2

y=-2/5x-2+1

y=-2/5-1

Hence, this is the required solution. I hope you find it helpful!

~SparklingFlower

--------------------------------------------------------------------------------------------------------------

Can someone please help me

Answers

Step-by-step explanation:

it will be around $90.86

Answer:

No. of adults who answered 'yes'

= 20% × 1073

= 214.6

Total amount of money paid

= (18%×$77)+$77

= $13.86+$77

= $90.86

What is the equation for the line in slope-intercept form?

Answers

Y=-8/2x+5
Hgfxdffggggujjcfccchhjbvg

10. If pointer 1 is spun and then pointer 2 is spun, determine the probability of landing on a color other than red on both spins. The probability is (Type an integer or a simplified fraction.)​

Answers

Answer:

Answer below

Step-by-step explanation:

1/8 is correct

in pic

_________________________________________

(Hope this helps can I pls have brainlist (crown)☺️)

Which expressions are equivalent to (1/3x+2x−5/3x)−(−1/3x+5) ? Select all correct expressions. −4x+5+3x 5−x 4x−5−3x x−5

Answers

Answer:

its x-5 and 4x-5-3x

Step-by-step explanation:

I took the test

Help pls I have no clue what this is

Answers

Answer:

  b = 76°

Step-by-step explanation:

Angle JIK is the smaller acute angle in one of four congruent right triangles. Angle GHK is the larger acute angle in those triangles, marked as 60°. The sum of the acute angles in a right triangle is 90°, so you have ...

  60° +(b-46)° = 90°

  14° +b = 90° . . . . . . . simplify

  b = 76° . . . . . . . subtract 14°

135 rational or irrational ​

Answers

Answer:

Rational

Step-by-step explanation:

I hope this helps!

It is an Irrational number

12
rectia
14. Ne
10.) Lars has a current annual salary of $38,650.
If he gets a raise of $3,280, what percent is
the raise of his current annual salary? Round
to the nearest whole percent.
(1) 6%
(2) 7%
2 %
8%
(4) 9%
10%

Answers

Answer:

3,280/38650 x 100% = 8%

The perimeter of a rectangle is 40 inches. If the width of the rectangle is 7 inches, what is the length?

Answers

Answer:

l=13in

Step-by-step explanation:

hope this helps

Answer:

13

Step-by-step explanation:

(40-7*2)/2=20-7=13

What is the surface of this shape

Answers

Answer:

Find the area of all of the different shapes (2D)

and then add them up together to find the surface area.

3×2

6×2

6×2

5×3

6×3

1×3

1×4

1×4

4×3

Then you add them up together.

6+12+12+15+18+3+4+4+12=ANS.

solve pls brainliest

Answers

Answer:

1. -64

2.-27

Step-by-step explanation:

Hopes this helps

:)

For a certain company, the cost function for producing x items is C(x)=30x+150 and the revenue function for selling x items is R(x)=−0.5(x−90)2+4,050. The maximum capacity of the company is 130 items.
The profit function P(x) is the revenue function R(x) (how much it takes in) minus the cost function C(x) (how much it spends). In economic models, one typically assumes that a company wants to maximize its profit, or at least make a profit!



Answers to some of the questions are given below so that you can check your work.



Assuming that the company sells all that it produces, what is the profit function?
P(x)=
Preview Change entry mode .

Hint: Profit = Revenue - Cost as we examined in Discussion 3.

What is the domain of P(x)?
Hint: Does calculating P(x) make sense when x=−10 or x=1,000?

The company can choose to produce either 60 or 70 items. What is their profit for each case, and which level of production should they choose?
Profit when producing 60 items =
Number


Profit when producing 70 items =
Number


Can you explain, from our model, why the company makes less profit when producing 10 more units?

Answers

Profit, revenue and cost are related, and can be calculated from one another.

The profit function is [tex]\mathbf{P(x) =-0.5(x -90)^2 - 30x + 3900}[/tex]The domain of the profit function is x > 0The profit when producing 60 items is 1650The profit when producing 70 items is 1600

The cost function is given as:

[tex]\mathbf{C(x) =30x + 150}[/tex]

The revenue function is given as:

[tex]\mathbf{R(x) =-0.5(x -90)^2 + 4050}[/tex]

(a) Calculate the profit function

This is calculated using:

[tex]\mathbf{P(x) = R(x) - C(x)}[/tex]

So, we have:

[tex]\mathbf{P(x) =-0.5(x -90)^2 + 4050 - 30x - 150}[/tex]

Evaluate like terms

[tex]\mathbf{P(x) =-0.5(x -90)^2 - 30x + 3900}[/tex]

(b) The domain of the profit function

When profit is 0 or less, then it becomes no profit.

Hence, the domain of the profit function is x > 0

(c) The profit for 60 and 70 items

Substitute 60 and 70 for x in P(x)

[tex]\mathbf{P(60) =-0.5(60 -90)^2 - 30 \times 60 + 3900}[/tex]

[tex]\mathbf{P(60) =1650}[/tex]

The profit when producing 60 items is 1650

[tex]\mathbf{P(70) =-0.5(70 -90)^2 - 30 \times 70 + 3900}[/tex]

[tex]\mathbf{P(70) = 1500}[/tex]

The profit when producing 70 items is 1600

(d) Why producing 10 more units less profit

When a function reaches the optimal value, the value of the function begins to reduce.

This means that, producing 10 more units takes the profit function beyond its maximum point.

Read more about profit, cost and revenue at:

https://brainly.com/question/11384352

Help help help help hell

Answers

Answer:

m = N - f

Step-by-step explanation:

The original equation:

N = m + f

if we have to solve for m, then we have to isolate it, so subtract f from both the sides and that would be the answer.

Hope this helps!

(a) A box contains 7 red balls, 6 white balls, and 7 black balls. Two balls are drawn at random from the box (with replacement of the first before the second is drawn). What is the probability of getting a red ball on the first draw and a white ball on the second

Answers

Answer:

Step-by-step explanation:

20=total marbles

  7/20 × 6/20=  21/400   or 5.25%

   red     white

Domain:
Range:
HELP RQ

Answers

i'm not too sure, as i'm learning this at the moment too. correct me if i'm wrong but wouldn't both be (infinity, infinity) ?? since there's arrows meaning its infinite

Which could be the first 4 terms of an arithmetic sequence ?

Answers

Awnser: B

Explain: if you line up and set up a graph with the numbers you will get b to be the awnser

(Can I get brainliest

-4√-32

answer it please ok thanks

Answers

Answer:

Step-by-step explanation:

4 √(-32) =

22.627417

A school wants to investigate the amount of exercise students do.
There are 1200 students at the school.
a Suggest a sensible number of students to be sampled.

Answers

Answer:

100-200     Step-by-step explanation: yes 600 is half the students most of them are going to be in class

A breakfast cereal producer makes its most popular product by combining just raisins and flakes in each box of cereal. The amounts of flakes in the boxes of this cereal are normally distributed with a mean of 370\,\text{g}370g370, start text, g, end text and a standard deviation of 24\,\text{g}24g24, start text, g, end text. The amounts of raisins are also normally distributed with a mean of 170\,\text{g}170g170, start text, g, end text and a standard deviation of 7\,\text{g}7g7, start text, g, end text.
Let T=T=T, equals the total amount of product in a randomly selected box, and assume that the amounts of flakes and raisins are independent of each other.
Find the probability that the total amount of product is less than 575\,\text{g}575g575, start text, g, end text.
You may round your answer to two decimal places.
P(T<575)\approx

Answers

Using the normal distribution, it is found that there is a 0.9192 = 91.92% probability that the total amount of product is less than 575 g.

In a normal distribution with mean [tex]\mu[/tex] and standard deviation [tex]\sigma[/tex], the z-score of a measure X is given by:

[tex]Z = \frac{X - \mu}{\sigma}[/tex]

It measures how many standard deviations the measure is from the mean.  After finding the z-score, we look at the z-score table and find the p-value associated with this z-score, which is the percentile of X.When two normal variables are added, the mean is the sum of the means while the standard deviation is the square root of the sum of the variances.

In this problem, the product is composed by flakes and raisins, and we have that:

[tex]\mu_F = 370, \sigma_F = 24, \mu_R = 170, \sigma_R = 7[/tex]

Hence, the distribution for the total amount of product has mean and standard deviation given by:

[tex]\mu = \mu_F + \mu_R = 370 + 170 = 540[/tex]

[tex]\sigma = \sqrt{\sigma_F^2 + \sigma_R^2} = \sqrt{24^2 + 7^2} = 25[/tex]

The probability that the total amount of product is less than 575 g is the p-value of Z when X = 575, hence:

[tex]Z = \frac{X - \mu}{\sigma}[/tex]

[tex]Z = \frac{575 - 540}{25}[/tex]

[tex]Z = 1.4[/tex]

[tex]Z = 1.4[/tex] has a p-value of 0.9192.

0.9192 = 91.92% probability that the total amount of product is less than 575 g.

A similar problem is given at https://brainly.com/question/22934264

Complete the table for the provided graph.

Answers

Answers on following order: 5, 2, 4, 2, 8, 0

Need helpPlzzzzzzzzzzzzzzzz

Answers

Answer:

2

Step-by-step explanation:

Is (1,5) a solution to this system of equations?
y=

3x+7
y=3x+2

Answers

Answer:

no, there is no solution

Step-by-step explanation:

put it in desmos, but the slopes are the same, so they are parallel and never touch. So, no solution.

8.) The grocery store is having a sale on frozen vegetables. 4 bags are being sold for $11.96. At this rate, what is the cost of:
a) 1 bag
b) 9 bags

Answers

a) 1 bag is $2.99

b) 9 bags are $26.91.

$11.96/4 is 2.99

So one bag costs $2.99 and for any rest of number of bags we just multiply that number by 2.99!!

Hope this helps. :)

Answer:

a) $2.99

b) $26.91

Step-by-step explanation:

Since 4 bags cost $11.96, you can find the price of an individual bag by dividing $11.96 by 4.

Since

$11.96 / 4 = $2.99

One bag costs $2.99

To find the price of 9 bags, just multiply 2.99$ by 9:

$2.99 * 9 = $26.91

a simple random sample of 60 items resulted in a sample mean of 80.

Answers

Answer:

(c) The effect of a larger sample size on the margin of error is that the margin of error will increase.

Step-by-step explanation:

H e l p this is confusing

Answers

Answer:

The answer would be 780 miles

Step-by-step explanation:

The reason that that's the answer because since it says that they will average about 65 miles per hour on a 12 hour trip, you have to multiply 65 time 12, which is 780 miles, so

65 miles x 12 hours = 780 miles   :)

Hope this helped!

What value is equivalent to 5 + [5 (4 + 2)]?
35
45
60
0110
Help pls thx

Answers

45 is equivalent to 5+[5(4+2)]

Answer:

45 5+[5(4+2)]?

Step-by-step explanation:

i hope is helped:)

Find the slope of the line
A -3
B 3
C -2
D 2​

Answers

Y=-2x-10
So it would be -2
Other Questions
f(x)= -2x^3 +14x -8g(x)= 2x+6 In the cycle shown is mantle made of silicate rocks why. Commercially prepared cloning vectors such as pUC18 are designed to contain several useful features. An example of one of these features is ________. Females are most likely to be stereotyped as __________, whereas males are most likely to be stereotyped as __________. what metaphor most closely describes the nature of space-time?a. chemistry formulab.fabricc. sine waved. moebius strip I am having a hard time on this question. Can you please explain it to me? Please keep it simple so I can understand it. I have already checked them and I just have this feeling that i'm not correct.John is going to a gym. He must pay a one time membership fee of $75 and a monthly fee of $30. let m=the number of months john has his membership. write an expression to represent this situation. how much will john pay for 6 months of membership? Leah's bill for breakfast at a restaurant was $68. She left an 18% tip. What was the amount of the tip? Hi, can someone help me with this. I need to submit this tonight. Help please...Question : Hi Suriana, How are you? I hope you and your family are in the pink of health. I have great news. My friends and I will be climbing Mount Kinabalu next week. I'm really nervous about climbing it. You enjoy climbing mountains too, don't you, Suriana? Could you please give me some advice about it? Hope to hear from you soon. Mei Li.In about 80 words write an email to Mei Li giving some advice. the minnesota vikings are going to win the game today said josh, a fifth grade fanThere are 8 improvements to be made. Which of the following statements is a correct implication of the author's argument? what is the estimate of 45.15 Monday, 11/8/2021In what way are you strong? which of the following are you using when you choose a sign based on meaning rather than a direct translation of the word concept mapping concept switching conceptual accuracy*** conceptual standardization PLEASE ANSWER ASAP!!!!!!!!!!The Edict of Milan only guarantees tolerance of Christianity, not recognition by thestate. Why did Constantine and Licinius take this position? Sa Love Ano ang importante sayo love o family? What should we do to protect the environment? Plzzzzz help meeee plzzzz A gopher lives in an underground burrow. It starts at a certain point, p, underground and digs straight down to triple its distance underthe surface. It then climbs straight down another 2 feet. Its final position is deeper than 8 feet under the surface.What was the gopher's original position, p, relative to the surface?Enter your answer in the box to complete the inequality. 1. Find the slope of the line that passes throughthe points (6, 13) and (-3, 7). Which group threatened British tax officials and burned effigies of British officials as a form of protest?